Forum: Offtopic Widerstandsnetzwerk berechnen


von Eric H. (eric_h)


Angehängte Dateien:

Lesenswert?

Hallo Zusammen,

hat jemand eine Idee wie diese Aufgabe zu lösen ist?
Mir fehlt da absolut der Ansatz.

Danke

Gruß Eric

von D. I. (Gast)


Lesenswert?

Eric H. schrieb:
> Mir fehlt da absolut der Ansatz.

Der Ansatz lautet Parallel- und Reihenschaltung von Widerständen. Viel 
Erfolg.

von Ben B. (Firma: Funkenflug Industries) (stromkraft)


Lesenswert?

Rg= 12V/0.6A= 20 Ohm
damit R= 32 Ohm
U R3 sind dann 2,4V

wenn ich mich nicht ohne Notizen vertan habe...

von Luca E. (derlucae98)


Lesenswert?

Tipp zur Vorgehensweise:

R2 und R3 zusammenfassen, R23 und R4 zusammenfassen, R234 und R1 
zusammenfassen und zuletzt R1234 und R5 zusammenfassen.

Spannung an R3:

Strom durch R1 berechnen (U_ges / R1234),
Spannung an R1 berechnen, U23 bzw U4 ist dann U_ges-U1, Strom durch R2 
bzw R3 berechnen (U23/R23).
Der Rest ist ohmsches Gesetz.

von Yalu X. (yalu) (Moderator)


Angehängte Dateien:

Lesenswert?

Man kann das Ganze auch grafisch veranschaulichen und erspart sich
dadurch das Aufstellen und Lösen von Gleichungen (s. Anhang). Da man
dazu ein Bildchen malen muss, ist diese Methode in Summe zwar nicht
unbedingt schneller, dafür aber unterhaltsamer :)

Erläuterung:

Jeder Widerstand ist durch ein Rechteck dargestellt. Dessen Breite
entspricht dem Strom I und dessen Höhe der Spannung U. Der
Widerstandswert selbst ist somit das Seitenverhältnis U/I. Bei einer
Serienschaltung werden die Teilschaltungen übereinandergestapelt
(gleicher Strom), bei einer Parallelschaltung werden sie nebeneinander
platziert (gleiche Spannung).

Da alle Widerstände den gleiche Wert haben sollen, haben die einzelnen
Rechtecke alle das gleiche Seitenverhältnis. Dewr Einfachheit halber
sind hier die Ströme und Spannungen so skaliert, dass die Widerstände
Quadrate sind.

Beim Zeichnen beginnt man mit den Widerständen, die bzgl. Serien- und
Parallelschaltungen am tiefsten verschachtelt liegen. Das sind hier R2
und R3. Diese werden übereinandergestapelt, dann der Parallelwiderstand
R4 seitlich angesetzt. Oben auf das Ganze kommt R1 und schließlich noch
R5 daneben.

So entsteht ein Gesamtrechteck, dass dem Gesamtwiderstand der Schaltung
entspricht. Dieses Rechteck hat die Breite 600 mA (der Gesamtstrom) und
die Höhe 12V (die Gesamtspannung). Da das Rechteck aus 8 × 5 kleinen
Quadraten zusammengesetzt ist, entfallen auf jedes dieser Quadrate 600mA
/ 8 = 75mA und 12V / 5 = 2,4V. Diese Werte gelten insbesondere auch für
R2 und R3. Deren Widerstand ist demzufolge 2,4V / 75mA = 32Ω. Für R4
sind Spannung und Strom doppelt, für R1 dreimal und für R5 fünfmal so
hoch.

Sehr schön wird in diesem Diagramm auch die Verlustleistung der
einzelnen Widerstände und Teilschaltungen visualisiert, diese entspricht
nämlich gerade der Fläche U·I der einzelnen Rechtecke.

von Timm T. (Gast)


Lesenswert?

Yalu X. schrieb:
> Da man
> dazu ein Bildchen malen muss, ist diese Methode in Summe zwar nicht
> unbedingt schneller, dafür aber unterhaltsamer :)

Das ist ja mal geil. Mit welcher Software hast Du das gemalt?

von Yalu X. (yalu) (Moderator)


Lesenswert?

Timm T. schrieb:
> Das ist ja mal geil. Mit welcher Software hast Du das gemalt?

Mit Inkscape. Das würde man aber mit fast jedem anderen Zeichentool
genauso gut hinbekommen.

In diesem Zusammenhang fällt mir gerade ein, dass ich hier im Forum eine
Knobelaufgabe gestellt habe, deren verschärfte Variante bis heute keiner
der Forenteilnehmer gelöst hat:

  Beitrag "Widerstandsknobelei"

Die Aufgabe ist zugegebenermaßen ziemlich schwer, man kann aber die
Lösung im Internet finden, wenn man nicht nach "Widerstandsnetzwerk"
sucht :)

von Joe F. (easylife)


Lesenswert?

Yalu X. schrieb:
> In diesem Zusammenhang fällt mir gerade ein, dass ich hier im Forum eine
> Knobelaufgabe gestellt habe, deren verschärfte Variante bis heute keiner
> der Forenteilnehmer gelöst hat

Welche "verschärfte Variante" meinst du genau?
Martin hat ja das Netzwerk im Prinzip gefunden 
(Beitrag "Re: Widerstandsknobelei")
Meinst du jetzt, auszurechnen, wie groß der Widerstand genau sein muss, 
damit man auf 1 Ohm Gesamtwiderstand kommt (also so um die 1.1312 Ohm je 
R...), und das als genauen Wert?

von Yalu X. (yalu) (Moderator)


Lesenswert?

Joe F. schrieb:
> Welche "verschärfte Variante" meinst du genau?
> Martin hat ja das Netzwerk im Prinzip gefunden

Martin hat eine Lösung für die einfache Variante gefunden, bei der der
resultierende Gesamtwiderstand egal ist.

In der verschärften Variante sollen sowohl die Einzelwiderstände als
auch der Gesamtwiderstand jeweils 1 Ω betragen und nach wie vor die
Ströme durch die Einzelwiderstände alle verschieden sein.

Für weitere Diskussionen zu dieser Aufgabe darf der andere Thread gerne
leichengefleddert werden :)

von Joe F. (easylife)


Lesenswert?

Yalu X. schrieb:
> In der verschärften Variante sollen sowohl die Einzelwiderstände als
> auch der Gesamtwiderstand jeweils 1 Ω betragen und nach wie vor die
> Ströme durch die Einzelwiderstände alle verschieden sein.

Ah ok, verstehe. Das macht es natürlich etwas interessanter... ;-)
Und mit 1 Ohm Gesamtwiderstand meinst du auch genau 1-komma-periode-Null 
Ohm?

: Bearbeitet durch User
von Yalu X. (yalu) (Moderator)


Lesenswert?

Joe F. schrieb:
> Und mit 1 Ohm Gesamtwiderstand meinst du auch genau 1-komma-periode-Null
> Ohm?

Natürlich. Da das ganze Rätsel maximal praxisfern ist, sind es auch die
gestellten Anforderungen. Dafür darfst du aber auch davon ausgehen, dass
die verwendeten 1Ω-Widerstände absolut perfekt sind :)

Alles Weitere zu dieser 1Ω-Knobelei ab jetzt aber bitte nur noch in den
Originalthread posten:

  Beitrag "Widerstandsknobelei"

von J.-u. G. (juwe)


Lesenswert?

Ben B. schrieb im Beitrag #4730461:
> Welcher kellerdoofe Saftsack bewertet eigentlich die Lösung mit -1 ...?

Ich auch nicht (da ich seit geraumer Zeit die Bewertungsfunktion 
grundsätzlich nicht verwende).

Es ist allerdings anzumerken, dass Eric nicht nach der Lösung seiner 
Hausaufgabe gefragt hat. Die richtige Antwort auf seine Frage wurde von 
D.I., im Beitrag über Deinem, gegeben.

Möglicherweise spielte dieser Sachverhalt bei den Überlegungen des einen 
oder anderen Bewerters eine Rolle.

von Eddy C. (chrisi)


Lesenswert?

Eric H. schrieb:
> hat jemand eine Idee wie diese Aufgabe zu lösen ist?
> Mir fehlt da absolut der Ansatz.

Du musst das Bild drehen. Den Winkel musst Du aber selbst herausfinden 
;-)

von Xeraniad X. (xeraniad)


Angehängte Dateien:

Lesenswert?

Guten Tag

Dies neue Elaborat enthält mögicherweise noch Tippfehler, ich bitte um 
Nachsicht.

Für mich ist der Übergang von konzentrierten zu verteilen Elementen 
faszinierend. Deshalb veruchte ich hier, einen generalisierten, auf 
Rekursion beruhenden Weg zu beschreiten, welcher die Türe zu einem 
Grenzübergang "n -> infty" öffnen kann.

schönen Tag
xeraniad

von Yalu X. (yalu) (Moderator)


Lesenswert?

Xeraniad X. schrieb:
> Deshalb veruchte ich hier, einen generalisierten, auf
> Rekursion beruhenden Weg zu beschreiten, welcher die Türe zu einem
> Grenzübergang "n -> infty" öffnen kann.

Ja, der Zusammenhang mit der Fibonacci-Folge und dem Goldenen Schnitt
beim Hinzufügen zusätzlicher Widerstände nach vorgegebenen Schema ist
interessant. Ich habe bei meinem ersten Beitrag hier schon überlegt,
eine entsprechende Teilaufgabe b (die im ursprünglichen Aufgabentext ja
ausgespart ist :)) zu formulieren.

Die Lösung kann entweder über Potenzen von Matrizen (wie bei dir) oder
Kettenbrüche erfolgen.

Um das Ganze grafisch zu visualisieren, kann man auch mein obiges
Diagramm mit den Quadraten heranziehen. Dort entspricht die Erweiterung
um weitere Widerstände dem Anfügen weiterer Quadrate an das bestehende
Gesamtrechteck, und zwar immer abwechselnd von oben und von rechts. Die
längere Seite des neuen Rechtecks ist damit jeweils die Summe der beiden
Seiten des vorherigen Rechtecks. Da die ganze Angelegenheit mit einem
1×1-Quadrat startet, wird sofort klar, dass die Seitenlängen des
anwachsenden Gesamtrechtecks jeweils einem Paar aufeinanderfolgender
Fibonacci-Zahlen, also (1,1), (1,2), (2,3), (3,5), (5,8), (8,13) usw.
entsprechen, und dass deren Verhältnis für n→∞ gegen das Verhältnis des
Goldenen Schnitts ½(1+√5) strebt.

von Joe F. (easylife)


Lesenswert?

Yalu X. schrieb:
> oder
> Kettenbrüche erfolgen.

Der Kettenbruch ist halt wesentlich einfacher von Hand zu lösen ;-)

von Xeraniad X. (xeraniad)


Lesenswert?

Danke Yalu für Deine Antwort, welche ich glaube verstanden zu haben.
Vermutlich werde ich demnächst, sofern es der Kalender erlaubt, die 
Ströme durch die Widerstände in der Kette von links her (vorzugsweise 
erneut mit Bezug auf F-Zahlen) formal zu ermitteln und damit die 
Leistungen angeben zu können, welche dann den Rechteck -Flächen für die 
Widerstände in der von Dir geposteten Graphik entsprechen sollten.
Wenn es dann gelingt, den gefundenen Zusammenhang betreffs Leistungen 
mit den Flächen und einer Fibonacci-Zahlen-Identitiät zu verifizieren, 
dann wird dies bestimmt erfreulich sein.

schönen Abend

Bitte melde dich an um einen Beitrag zu schreiben. Anmeldung ist kostenlos und dauert nur eine Minute.
Bestehender Account
Schon ein Account bei Google/GoogleMail? Keine Anmeldung erforderlich!
Mit Google-Account einloggen
Noch kein Account? Hier anmelden.